Is the question flawed

This topic has expert replies
Legendary Member
Posts: 712
Joined: Fri Sep 25, 2015 4:39 am
Thanked: 14 times
Followed by:5 members

Is the question flawed

by Mo2men » Mon Feb 27, 2017 12:55 am
If -6 < k < 6, is k > 1 ?

(1) 1/k > 1
(2) -k < -2

S1) 1/k >1 implies that 1/k is Positive. To be larger than 1, is must be fraction.

than multiply by k..... K<1. Answer is No.

Sufficient

S2 ) -k < -2....k>2>1 then Answer is yes

Sufficient

The answer should be D. However, both answers are contradicting each other. Is it allowable in GMAT? is the question flawed?

Thanks

User avatar
GMAT Instructor
Posts: 15539
Joined: Tue May 25, 2010 12:04 pm
Location: New York, NY
Thanked: 13060 times
Followed by:1906 members
GMAT Score:790

by GMATGuruNY » Mon Feb 27, 2017 3:41 am
If -6 < k < 6, is k > 1 ?

(1) 1/k > 1
(2) -k < -2
Statement 1 implies that 0 < k < 1, while Statement 2 indicates that k > 2.
Since the two statements contradict each other, the problem seems flawed.
Also, the constraint in red seems irrelevant.
Each statement on its own is sufficient to determine whether k > 1, regardless of the constraint in red.
Private tutor exclusively for the GMAT and GRE, with over 20 years of experience.
Followed here and elsewhere by over 1900 test-takers.
I have worked with students based in the US, Australia, Taiwan, China, Tajikistan, Kuwait, Saudi Arabia -- a long list of countries.
My students have been admitted to HBS, CBS, Tuck, Yale, Stern, Fuqua -- a long list of top programs.

As a tutor, I don't simply teach you how I would approach problems.
I unlock the best way for YOU to solve problems.

For more information, please email me (Mitch Hunt) at [email protected].
Student Review #1
Student Review #2
Student Review #3

GMAT/MBA Expert

User avatar
Elite Legendary Member
Posts: 10392
Joined: Sun Jun 23, 2013 6:38 pm
Location: Palo Alto, CA
Thanked: 2867 times
Followed by:511 members
GMAT Score:800

by [email protected] » Mon Feb 27, 2017 1:17 pm
Hi Mo2men,

Your evaluation of this question is correct. On the Official GMAT, each of the two Facts will 'point' to the actual answer to the given question (although they might 'point' to a different answer as well - and it's your job to prove whether the answer is consistent or inconsistent). However, the two Facts will NEVER contradict one another in the way that they do in this question. What is the source of this prompt?

GMAT assassins aren't born, they're made,
Rich
Contact Rich at [email protected]
Image

GMAT/MBA Expert

User avatar
GMAT Instructor
Posts: 3008
Joined: Mon Aug 22, 2016 6:19 am
Location: Grand Central / New York
Thanked: 470 times
Followed by:34 members

by Jay@ManhattanReview » Mon Feb 27, 2017 8:26 pm
Mo2men wrote:If -6 < k < 6, is k > 1 ?

(1) 1/k > 1
(2) -k < -2

S1) 1/k >1 implies that 1/k is Positive. To be larger than 1, is must be fraction.

than multiply by k..... K<1. Answer is No.

Sufficient

S2 ) -k < -2....k>2>1 then Answer is yes

Sufficient

The answer should be D. However, both answers are contradicting each other. Is it allowable in GMAT? is the question flawed?

Thanks
Hi Mo2men,

Your assessment is correct; the problem is flawed. I agree with Rich's and Mitch's insight.

An official DS question comprising the prompt, statement 1 and statement 2 demonstrates a holistic situation. A result from a statement must not contradict that from the other statement. If the answer to the questions is D for a "Yes/No" type of question, and statement 1 renders "Yes," statement 2 must also render "Yes;" and if statement 1 renders "No," statement 2 must also render "No."

In this question, we have two issues: 1. Answer from statement 1 is "No," while that from statement 2 is "Yes." This is not possible in an official DS question. 2. Mitch rightly pointed out that there is no bearing of the information -6 < k < 6 given in the prompt. It's redundant. An official question will not have this scenario.

Hope this helps!

Find great DS questions here: Manhattan Review GMAT Data Sufficiency Guide

-Jay
_________________
Manhattan Review GMAT Prep

Locations: New York | Bangkok | Abu Dhabi | Rome | and many more...

Schedule your free consultation with an experienced GMAT Prep Advisor! Click here.

GMAT Instructor
Posts: 2630
Joined: Wed Sep 12, 2012 3:32 pm
Location: East Bay all the way
Thanked: 625 times
Followed by:119 members
GMAT Score:780

by Matt@VeritasPrep » Fri Mar 03, 2017 3:47 am
It's possible that the first statement was meant to be 1/k < 1: it's so easy to accidentally flip a sign and miss the mistake when writing/editing a batch of math problems.